Stay ahead of learning milestones! Enroll in a class over the summer!

Contests & Programs AMC and other contests, summer programs, etc.
AMC and other contests, summer programs, etc.
3 M G
BBookmark  VNew Topic kLocked
Contests & Programs AMC and other contests, summer programs, etc.
AMC and other contests, summer programs, etc.
3 M G
BBookmark  VNew Topic kLocked
G
Topic
First Poster
Last Poster
Gheorghe Țițeica 2025 Grade 9 P2
AndreiVila   5
N an hour ago by sqing
Source: Gheorghe Țițeica 2025
Let $a,b,c$ be three positive real numbers with $ab+bc+ca=4$. Find the minimum value of the expression $$E(a,b,c)=\frac{a^2+b^2}{ab}+\frac{b^2+c^2}{bc}+\frac{c^2+a^2}{ca}-(a-b)^2.$$
5 replies
AndreiVila
Mar 28, 2025
sqing
an hour ago
Geometry Parallel Proof Problem
CatalanThinker   4
N an hour ago by CatalanThinker
Source: No source found, just yet, please share if you find it though :)
Let M be the midpoint of the side BC of triangle ABC. The bisector of the exterior angle of point A intersects the side BC in D. Let the circumcircle of triangle ADM intersect the lines AB and AC in E and F respectively. If the midpoint of EF is N, prove that MN || AD.
I have done some constructions, but still did not quite get to the answer, see diagram attached below
4 replies
CatalanThinker
2 hours ago
CatalanThinker
an hour ago
Find all functions $f$ is strictly increasing : \(\mathbb{R^+}\) \(\rightarrow\)
guramuta   5
N an hour ago by ja.
Find all functions $f$ is strictly increasing : \(\mathbb{R^+}\) \(\rightarrow\) \(\mathbb{R^+}\) such that:
i) $f(2x)$ \(\geq\) $2f(x)$
ii) $f(f(x)f(y)+x) = f(xf(y)) + f(x) $
5 replies
guramuta
Yesterday at 1:45 PM
ja.
an hour ago
Hard combi
EeEApO   3
N an hour ago by CatalanThinker
In a quiz competition, there are a total of $100 $questions, each with $4$ answer choices. A participant who answers all questions correctly will receive a gift. To ensure that at least one member of my family answers all questions correctly, how many family members need to take the quiz?

Now, suppose my spouse and I move into a new home. Every year, we have twins. Starting at the age of $16$, each of our twin children also begins to have twins every year. If this pattern continues, how many years will it take for my family to grow large enough to have the required number of members to guarantee winning the quiz gift?
3 replies
EeEApO
Yesterday at 6:08 PM
CatalanThinker
an hour ago
Is this FE is solvable?
ItzsleepyXD   1
N an hour ago by jasperE3
Source: Own , If not appear somewhere before
Find all function $f : \mathbb{R} \to \mathbb{R}$ such that for all $x,y \in  \mathbb{R}$ . $$f(x+f(y))+f(x+y)=2x+f(y)+f(f(y))$$. Original
1 reply
ItzsleepyXD
4 hours ago
jasperE3
an hour ago
Equilateral triangle formed by circle and Fermat point
Mimii08   1
N 2 hours ago by srirampanchapakesan
Source: Heard from a friend
Hi! I found this interesting geometry problem and I would really appreciate help with the proof.

Let ABC be an acute triangle, and let T be the Fermat (Torricelli) point of triangle ABC. Let A1, B1, and C1 be the feet of the perpendiculars from T to the sides BC, AC, and AB, respectively. Let ω be the circle passing through points A1, B1, and C1. Let A2, B2, and C2 be the second points where ω intersects the sides BC, AC, and AB, respectively (different from A1, B1, C1).

Prove that triangle A2B2C2 is equilateral.

1 reply
Mimii08
Yesterday at 10:36 PM
srirampanchapakesan
2 hours ago
Inspired by Kosovo 2010
sqing   0
2 hours ago
Source: Own
Let $ a,b>0  , a+b\leq k $. Prove that
$$\left(1+\frac{1}{a(b+1)}\right)\left(1+\frac{1}{b(a+1)}\right)\geq\left(1+\frac{4}{k(k+2)}\right)^2$$$$\left(1+\frac {a}{b(a+1)}\right)\left(1+\frac {b}{a(b+1)}\right) \geq\left(1+\frac{2}{k+2}\right)^2$$Let $ a,b>0  , a+b\leq 2 $. Prove that
$$\left(1+\frac{1}{a(b+1)}\right)\left(1+\frac{1}{b(a+1)}\right)\geq \frac{9}{4} $$$$\left(1+\frac {a}{b(a+1)}\right)\left(1+\frac {b}{a(b+1)}\right) \geq \frac{9}{4} $$
0 replies
sqing
2 hours ago
0 replies
Kosovo MO 2010 Problem 5
Com10atorics   20
N 2 hours ago by sqing
Source: Kosovo MO 2010 Problem 5
Let $x,y$ be positive real numbers such that $x+y=1$. Prove that
$\left(1+\frac {1}{x}\right)\left(1+\frac {1}{y}\right)\geq 9$.
20 replies
Com10atorics
Jun 7, 2021
sqing
2 hours ago
Fourth powers and square roots
willwin4sure   39
N 3 hours ago by awesomeming327.
Source: USA TSTST 2020 Problem 4, by Yang Liu
Find all pairs of positive integers $(a,b)$ satisfying the following conditions:
[list]
[*] $a$ divides $b^4+1$,
[*] $b$ divides $a^4+1$,
[*] $\lfloor\sqrt{a}\rfloor=\lfloor \sqrt{b}\rfloor$.
[/list]

Yang Liu
39 replies
willwin4sure
Dec 14, 2020
awesomeming327.
3 hours ago
Interesting inequalities
sqing   1
N 3 hours ago by sqing
Source: Own
Let $ a,b >0 $ and $ a^2-ab+b^2\leq 1 $ . Prove that
$$a^4 +b^4+\frac{a }{b +1}+ \frac{b }{a +1} \leq 3$$$$a^3 +b^3+\frac{a^2}{b^2+1}+ \frac{b^2}{a^2+1} \leq 3$$$$a^4 +b^4-\frac{a}{b+1}-\frac{b}{a+1} \leq 1$$$$a^4+b^4 -\frac{a^2}{b^2+1}- \frac{b^2}{a^2+1}\leq 1$$$$a^3+b^3 -\frac{a^3}{b^3+1}- \frac{b^3}{a^3+1}\leq 1$$
1 reply
sqing
3 hours ago
sqing
3 hours ago
AMC and JMO qual question
HungryCalculator   4
N Apr 22, 2025 by eyzMath
Say that on the AMC 10, you do better on the A than the B, but you still qualify for AIME thru both. Then after your AIME, it turns out that you didn’t make JMO through the A+AIME index but you did pass the threshold for the B+AIME index.

does MAA consider your B+AIME index over the A+AIME index and consider you a JMO qualifier even tho Your A test score was higher?

4 replies
HungryCalculator
Apr 17, 2025
eyzMath
Apr 22, 2025
AMC and JMO qual question
G H J
G H BBookmark kLocked kLocked NReply
The post below has been deleted. Click to close.
This post has been deleted. Click here to see post.
HungryCalculator
539 posts
#1
Y by
Say that on the AMC 10, you do better on the A than the B, but you still qualify for AIME thru both. Then after your AIME, it turns out that you didn’t make JMO through the A+AIME index but you did pass the threshold for the B+AIME index.

does MAA consider your B+AIME index over the A+AIME index and consider you a JMO qualifier even tho Your A test score was higher?
Z K Y
The post below has been deleted. Click to close.
This post has been deleted. Click here to see post.
bebebe
993 posts
#2
Y by
i think in this case you'll make jmo
Z K Y
The post below has been deleted. Click to close.
This post has been deleted. Click here to see post.
Bread10
94 posts
#3
Y by
Yes of course. A more interesting question would be if you DIDN'T qualify for AIME through B, then would you still be able to do it, which I believe the answer would be no.
This post has been edited 1 time. Last edited by Bread10, Apr 17, 2025, 12:47 AM
Z K Y
The post below has been deleted. Click to close.
This post has been deleted. Click here to see post.
mhgelgi
742 posts
#4
Y by
They have four separate cutoffs {10A + A1, 10A + A2, 10B + A1, 10B + A2}
whichever you succeed in doesn't matter.
Z K Y
The post below has been deleted. Click to close.
This post has been deleted. Click here to see post.
eyzMath
9 posts
#5
Y by
yeah i think so
Z K Y
N Quick Reply
G
H
=
a